Guadalupe models the volume of a popcorn box as a right rectangular prism and the box can hold 46 cubic inches of popcorn when it is full. Its length is 2 3 4 2 4 3 ​ in and its height is 7 1 2 7 2 1 ​ in. Find the width of the popcorn box in inches. Round your answer to the nearest tenth if necessary.

Answers

Answer 1

The width of the rectangular prism popcorn box is approximately 2.27 inches when rounded to the nearest tenth.

How to Find the Width of a Rectangular Prism?

The volume of a right rectangular prism is given by:

V = lwh

where V is the volume, l is the length, w is the width, and h is the height.

We are given that the box can hold 46 cubic inches of popcorn, the length is 2¾ inches, and the height is 7½ inches. Let's use w to represent the width we are trying to find.

So we have:

46 = (2¾)w(7½)

To solve for w, we can divide both sides of the equation by (2¾)(7½):

46 / ((2¾)(7½)) = w

Simplifying the right-hand side, we get:

w ≈ 2.27

Learn more about rectangular prism on:

https://brainly.com/question/23665595

#SPJ1

Complete Question:

Guadalupe models the volume of a popcorn box as a right rectangular prism and the box can hold 46 cubic inches of popcorn when it is full. Its length is 2¾ inches and its height is 7½ inches. Find the width of the popcorn box in inches. Round your answer to the nearest tenth if necessary.


Related Questions

Use the information in the table below to answer the following question. name of fund nav offer price upton group $18.47 $18.96 green energy $17.29 $18.01 tjh small-cap $18.43 $19.05 whi health $20.96 nl for which of the funds shown would you pay the most commission on the purchase of 100 shares? a. green energy b. tjh small-cap c. upton group d. whi health

Answers

WHI Health Fund pays the most commission on the purchase of 100 shares with a commission of $96.00. Thus, option d is correct.

Funds offer price for  Upton Group = $18.96 - $18.47

Funds offer price for Green Energy fund = $18.01 - $17.29

Funds offer price for TJH Small-Cap fund = $19.05 - $18.43

Funds offer price for WHI Health fund = $20.96 - $20.00

To calculate the commission on purchasing shares, we need to find the allowance between the price ranges and then multiply the value by 100.

For the Upton Group fund, Commission = (Offer price - NAV) * 100

= ($18.96 - $18.47) * 100

= $49.00

For the Green Energy fund, Commission = (Offer price - NAV) * 100

= ($18.01 - $17.29) * 100

= $72.00

For the TJH Small-Cap fund, Commission = (Offer price - NAV) * 100

= ($19.05 - $18.43) * 100

= $62.00

For the WHI Health fund, Commission = (Offer price - NAV) * 100

= ($20.96 - $20.00) * 100

= $96.00

Therefore, we can conclude that the WHI Health fund pays the most commission of $96.00.

To learn more about commission

https://brainly.com/question/12175457

#SPJ4

Determine Whether The Series Is Convergent Or Divergent. Σ ^n√14

Answers

Based on the Root Test, the series Σ^n√14 is convergent.

Hi! To determine if the series Σ^n√14 is convergent or divergent, we need to analyze the terms involved. The series can be written as:

Σ (n√14)

This is a sum of terms, where each term is the n-th root of 14, and we want to find out if the sum converges or diverges as n goes to infinity.

In this case, the series is a type of p-series, where the terms follow the general form of 1/n^p. To be a convergent p-series, p must be greater than 1. Here, the terms are in the form of 14^(1/n), which can be rewritten as (14^(1))^(-n) or 14^(-n). This is not a p-series, as the exponent is not in the form of 1/n^p.

To further analyze the series, we can use the Divergence Test. If the limit of the terms as n goes to infinity is not equal to zero, then the series is divergent. So, let's find the limit:

lim (n → ∞) (14^(-n))

As n approaches infinity, the exponent -n becomes increasingly negative, and 14^(-n) approaches 0. However, the Divergence Test is inconclusive in this case, as it only confirms divergence if the limit is not equal to zero.

To determine convergence or divergence, we can use the Root Test. The Root Test states that if the limit of the n-th root of the absolute value of the terms as n goes to infinity is less than 1, then the series converges. Let's find the limit:

lim (n → ∞) |(14^(-n))|^(1/n)

This simplifies to:

lim (n → ∞) 14^(-1)

Since 14^(-1) is a constant value less than 1, the limit is less than 1.

Thus, based on the Root Test, the series Σ^n√14 is convergent.

To learn more about root test, refer below:

https://brainly.com/question/30906556

#SPJ11

Which statement is true considering a significance level of 5%?

A. The result is statistically significant, which implies that wearing a watch does not help people manage their time better.

B. The result is not statistically significant, which implies that this result could be due to random chance.

C. The result is statistically significant, which implies that wearing a watch helps people manage their time better.

D. The result is not statistically significant, which implies that wearing a watch does not help people manage their time better.

Answers

Given the scenerio in the picture about corn, the statement that is true looking at a significance level of 5% is The result is not statistically significant which implies that spraying the corn plants with the new type of fertilizer does increase the growth rate.

What is the does the 5% significance level mean in the context provided?

Looking at the statement "The result is not statistically significant,"this means that the p-value (probability value) of the test was greater than 0.05. It could have 0.15 oe 0.2.

When a test is greater than 0.05 or 5 % significance level, it shows that the what is happening to the corn (increase in growth rate) could have been as a result of chance alone.

Find more exercises on 5 % significance level;

https://brainly.com/question/24095750

#SPJ1

12 less than the product of 3 and a number, x, is at most -18

Answers

The given inequality is 3x - 12 ≤ -18. To solve for x, we can add 12 to both sides of the inequality to obtain 3x ≤ -6. Then, dividing both sides of the inequality by 3 gives x ≤ -2. Therefore, any value of x less than or equal to -2 will satisfy the inequality.

In solving the inequality, we first used the addition property of inequalities to add 12 to both sides of the inequality. This property states that if a < b, then a + c < b + c, where c is any real number. By adding 12 to both sides, we were able to isolate the variable term on one side of the inequality.

Next, we used the division property of inequalities to divide both sides of the inequality by 3. This property states that if a < b and c > 0, then a/c < b/c. By dividing both sides of the inequality by 3, we were able to solve for x.

Finally, we found that any value of x less than or equal to -2 will satisfy the inequality. This means that the solution set for the inequality is {x | x ≤ -2}. We also verified that x = -2 is a valid solution to the inequality, which confirms our solution.

To know more about inequality refer here:

https://brainly.com/question/30231190#

#SPJ11

In 2010, Keenan paid $2,826 in federal income tax, which is 70% less than he paid in 2009. How much did he pay in 2009?

Answers

Based on the above, Keenan paid $9,420 in federal income tax in 2009.

What is the income tax?

Let X be the amount Keenan paid in taxes in 2009.

According to the problem, Keenan paid 70% less in 2010 than he did in 2009. This means that he paid only 30% of what he paid in 2009, since 100% - 70% = 30%.

We can express this mathematically as:

0.30X = 2,826

To solve for X, we can divide both sides of the equation by 0.30:

X = 2,826 ÷  0.30

X = 9,420

Therefore, Keenan paid $9,420 in federal income tax in 2009.

Learn more about income tax from

https://brainly.com/question/30157668

#SPJ1

Learning


Diagnostic


Analytics


Recommendations


Skill plans


Math


Language arts


Common Core


Sixth grade


P. 6 Compare and order rational numbers: word problems ETK


You have prizes to reveall Go


Manuel and his friends built model cars using pieces of wood and plastic wheels. They rolled


the cars down a ramp and measured to see whose car would coast the farthest. Manuel's car


coasted 10 feet, Richard's car coasted 10. 5 feet, and Diego's car coasted 10


2


feet.


6


How many of the cars coasted more than 10. 75 feet?


Submit

Answers

Number of cars that coasted more than 10.75 feet = 1

How many of the cars coasted more than 10.75 feet?

To solve this problem, you need to compare the distance each car coasted to 10.75 feet, which is the threshold for determining whether a car coasted more or less than 10.75 feet.

Manuel's car coasted 10 feet, which is less than 10.75 feet, so it did not coast more than 10.75 feet.

Richard's car coasted 10.5 feet, which is also less than 10.75 feet, so it did not coast more than 10.75 feet either.

Diego's car coasted 102 feet, which is more than 10.75 feet. Therefore, only one car coasted more than 10.75 feet, and the answer is 1.

So the answer is:

Number of cars that coasted more than 10.75 feet = 1

Learn more about the Number of cars

brainly.com/question/12603

#SPJ11

In triangle STU as equals 50 inches to equals 58 inches and you equals 27 inches find the measure of angle us to the nearest 10th of a degree

Answers

The measure of the angle ∠S, obtained using the law of cosines is about 59.4°

What is the law of cosines?

The law of cosines states that the square of the length of a side of a triangle is equivalent to the sum of the squares of the other two sides less the product of the length of the other two sides and the angle between them. Mathematically; a² = b² + c² - 2·b·c·cos(A)

Where;

a, b, and c = The length of the sides of the triangle

A = The angle between b and c

The lengths of the sides of the triangle, obtained from a similar triangle are;

s = 50 inches, t = 58 inches, and u = 27 inches

The measure of the angle ∠S is required

The angle ∠S is the angle facing TU or the side s

According to law of cosines, we get;

s² = t² + u² - 2·t·u·cos(∠S)

Therefore;

50² = 58² + 27² - 2 × 58 × 27 × cos(∠S)

2 × 58 × 27 × cos(∠S) = 58² + 27² - 50²

cos(∠S) = (58² + 27² - 50²)/(2 × 58 × 27)

∠S = cos((58² + 27² - 50²)/(2 × 58 × 27)) ≈ 59.4°

The measure of the angle ∠S is about 59.4°

Learn more on the law of cosines here: https://brainly.com/question/13409288

#SPJ4

Of students taking both English 12 Honors and a senior level math course (AP Stats, AP Calculus, Pre-Calculus, College Prep Math, or Topics), 37% of students got an A in English, and 24% of students got an A in Math. 16% got an A in both classes.



What is the probability that a randomly selected student got an A in Math, but not English?

Answers

The probability that a randomly selected student got an A in Math, but not English, is 8%

Let A be the event that a student got an A in Math, and B be the event that a student got an A in English. Then, we want to find P(A and not B), or the probability that a student got an A in Math, but not English.

We know that P(A and B) = 0.16, or the probability that a student got an A in both Math and English. We also know that P(B) = 0.37, or the probability that a student got an A in English. Therefore, the probability of a student getting an A in Math, given that they got an A in English, can be calculated using the formula for conditional probability:

P(A | B) = P(A and B) / P(B)

P(A | B) = 0.16 / 0.37

P(A | B) = 0.43

This means that the probability of a student getting an A in Math, given that they got an A in English, is approximately 0.43.

To find the probability of a student getting an A in Math, but not English, we can subtract the probability of getting an A in both classes from the probability of getting an A in Math:

P(A and not B) = P(A) - P(A and B)

P(A and not B) = 0.24 - 0.16

P(A and not B) = 0.08

Therefore, the probability that a randomly selected student got an A in Math, but not English, is 0.08 or 8%.

To learn more about probability : https://brainly.com/question/251701

#SPJ11

Your cousin is bulding a sandbox for his daughter. How much sand will he need to fill the box? Explain. How much paint will he need to paint all six surfaces of the sandbox? Explain. ​

Answers

The amount of paint that he will need to paint all six surfaces of the sandbox is: 68 square feet

How to find the volume of the prism?

Since the image is a rectangular prism

The volume of the box can be obtained by using the formula:

Volume = l * b * h

The box has a dimension of 1ft x 4ft x 6ft

The volume of the box = 1 x 4 x 6 = 24 cubic feet

Therefore, the volume of sand needed to fill the box will be = 24 cubic feet of sand

The surface area of the box can be obtained using the formula:

2(lb + lh + bh)

= 2(1*4 + 1*6 + 4*6)

=2(4 + 6 + 24)

=2 (34)

= 68 square feet

Therefore a total surface area of 68 square feet needs to be painted

Read more about Volume of box at: https://brainly.com/question/463363

#SPJ1

Complete question is:

Your cousin is building a Sandbox for his daughter how much sand will he need to fill the Box? Explain. How much paint will he need to paint all six surface of the sandbox? Explain. 1ft 4ft 6ft not answer choices

Use the appropriate compound interest formula to find the amount that will be in each account, given the stated conditions.
$24.000 invested at 4% annual interest for 7 years compounded (a) annually: (b) semiannually

Answers

Account amount after 7 years will be approximately:
(a) $31,950.42 when compounded annually
(b) $32,166.25 when compounded semiannually

How to calculate compound interest after 7 years?

We'll be using the compound interest formula to find the amount in each account for both (a) annual compounding and (b) semiannual compounding.

The compound interest formula is: A = P(1 + r/ⁿ)ⁿᵃ

Where:
A = the future amount in the account
P = the principal (initial investment)
r = Annual interest rate
n = Interest is compounded per year in numbers
a = the number of years

(a) Annual Compounding:
In this case n = 1.

P = $24,000
r = 4% = 0.04
n = 1
t = 7

A = 24000(1 + 0.04/1)¹ˣ⁷
A = 24000(1 + 0.04)⁷
A = 24000(1.04)⁷
A ≈ $31,950.42

(b) Semiannual Compounding:
For semiannual compounding, the interest is compounded twice a year, so n = 2.

P = $24,000
r = 4% = 0.04
n = 2
t = 7

A = 24000(1 + 0.04/2)²ˣ⁷
A = 24000(1 + 0.02)¹⁴
A ≈ $32,166.25

Learn more about annual interest rate.

brainly.com/question/30573341

#SPJ11

A line that passes through the point (x,y) with a y-intercept of b and a slope of m can be represented by the equation y = mx + b.
Joe drew a line on the coordinate plane that passes through the point (-10,52) and has a slope of -6.5. The y-intercept of the line is

Answers

The y-intercept of the line is -13.

To find the y-intercept of the line, we can use the slope-intercept form of the equation of a line: y = mx + b,

where m is the slope and b is the y-intercept.

Given that the line passes through the point (-10, 52) and has a slope of -6.5, we can substitute these values into the equation:

52 = -6.5(-10) + b

Simplifying the equation:

52 = 65 + b

To isolate b, we subtract 65 from both sides:

52 - 65 = b

Simplifying further:

b = -13

For similar question on y-intercept.

https://brainly.com/question/30239450

#SPJ11

What is the variance of the following set of data?


4, 44, 404, 244, 4, 74, 84, 64

Answers

The variance of the given data set is 18603.39.

To find the variance of the given data set {4, 44, 404, 244, 4, 74, 84, 64}, follow these steps:

Step 1: First, we need to find the mean of the data set:

Mean = (4 + 44 + 404 + 244 + 4 + 74 + 84 + 64) / 8 = 120.5

Step 2: Next, we calculate the deviation of each data point from the mean:

(4 - 120.5) = -116.5

(44 - 120.5) = -76.5

(404 - 120.5) = 283.5

(244 - 120.5) = 123.5

(4 - 120.5) = -116.5

(74 - 120.5) = -46.5

(84 - 120.5) = -36.5

(64 - 120.5) = -56.5

Step 3: Now, we square each deviation:

[tex](-116.5)^2 = 13556.25\\(-76.5)^2 = 5852.25\\(283.5)^2 = 80322.25\\(123.5)^2 = 15252.25\\(-116.5)^2 = 13556.25\\(-46.5)^2 = 2162.25 \\(-36.5)^2 = 1332.25\\(-56.5)^2 = 3192.25[/tex](-116.5)^2 = 13556.25

Step 4: We add up all the squared deviations:

13556.25 + 5852.25 + 80322.25 + 15252.25 + 13556.25 + 2162.25 + 1332.25 + 3192.25 = 130223.75

Step 5: We divide the sum of the squared deviations by the number of data points minus 1 to get the variance:

Variance = 130223.75 / 7 = 18603.39 (rounded to two decimal places)

Therefore, the variance of the data set is 18603.39.

To know more about Variance refer here:

https://brainly.com/question/14116780

#SPJ11


4 Find the value xa where the function is discontinuousFor the point of discontinuity, give (a) f) if it exists. (b) lm (0) Im . () im 16), and (ej identity which conditions for continuity are not man -O (Use a coma o separate answers as needed Select the choice below and necessary, tal in the trawer box within your choice ОА ка) OB) is undefined (b) Select the choice below and necessary, tu in the answer box within your chale ΟΑ. lim) OB lim does not exist

Answers

So, the answer is:
(a) f(x) does not exist at xa = 16.
(b) lim f(x) as x approaches 16 does not exist.
(c) None of the conditions for continuity are met at xa = 16.

To find the value of xa where the function is discontinuous, we need to look for any points where the function is undefined or where the left and right limits of the function are not equal.
(a) From the given information, we know that the function is undefined at xa = 16. So, this is the point of discontinuity.
(b) To find the left and right limits at xa = 16, we need to approach the point from both sides of the function. So,
lim f(x) as x approaches 16 from the left (denoted as lim-) = Im = 0
lim f(x) as x approaches 16 from the right (denoted as lim+) = Im = 16
Since the left and right limits are not equal, the limit as x approaches 16 does not exist. So,
lim f(x) as x approaches 16 (denoted as lim) does not exist.
(c) To determine which conditions for continuity are not met, we need to check if the function satisfies the three conditions for continuity at xa = 16.
i) The function must be defined at xa = 16. Since the function is undefined at xa = 16, this condition is not met.
ii) The left and right limits of the function must exist and be equal at xa = 16. Since the left and right limits are not equal, this condition is not met.
iii) The value of the function at xa = 16 must be equal to the limit of the function at xa = 16. Since the limit does not exist, this condition is also not met.
Therefore, none of the conditions for continuity are met at xa = 16.
So, the answer is:
(a) f(x) does not exist at xa = 16.
(b) lim f(x) as x approaches 16 does not exist.
(c) None of the conditions for continuity are met at xa = 16.
Note: The terms "discontinuous" and "continuity" are used throughout the explanation to describe the concept and the point of interest. The term "function" refers to the given function that we are analyzing.

Learn more about continuity Visit: brainly.com/question/24637240

#SPJ11

Triangle ABC has vertices A(3, 1), B(8, y), and C(4, 6). The area of the triangle is 12 square units. Y=? The perimeter of △ABC is ? Units. Round your answer to the nearest tenth of a unit

Answers

The value of y is 50, and the perimeter of triangle ABC is approximately 49.3 units.

How to find the value of y and the perimeter of a triangle given its vertices and area?

To find the value of y in the coordinate of vertex B, we can use the formula for the area of a triangle given the coordinates of its vertices:

Area =[tex]\frac{ 1}{2}[/tex] * |(x1(y2-y3) + x2(y3-y1) + x3(y1-y2))|

Let's substitute the given values into the formula:

12 = [tex]\frac{ 1}{2}[/tex]* |(3(y-6) + 8(6-1) + 4(1-y))|

Simplifying the equation:

24 = |(3y - 18 + 40 + 4 - 4y)|

24 = |(-y + 26)|

Now, we can solve the equation by considering both the positive and negative values of the absolute expression:

-y + 26 = 24

-y = -2

y = 2

-y + 26 = -24

-y = -50

y = 50

So we have two possible values for y: y = 2 or y = 50.

To determine the correct value for y, we need to analyze the given information further. Since we know that triangle ABC is not an isosceles triangle (as the base lengths differ), we can eliminate the possibility of y = 2, leaving us with y = 50.

Now, let's calculate the perimeter of triangle ABC using the coordinates of its vertices:

AB = [tex]\sqrt((8 - 3)^2 + (y - 1)^2)[/tex]

BC = [tex]\sqrt((4 - 8)^2 + (6 - y)^2)[/tex]

CA = [tex]\sqrt((3 - 4)^2 + (1 - 6)^2)[/tex]

Perimeter = AB + BC + CA

Substituting the known values:

Perimeter = [tex]\sqrt((8 - 3)^2 + (50 - 1)^2) + \sqrt((4 - 8)^2 + (6 - 50)^2) + \sqrt((3 - 4)^2 + (1 - 6)^2)[/tex]

Calculating each term:

Perimeter = [tex]\sqrt(25 + 2401) + \sqrt(16 + 2025) + \sqrt(1 + 25)[/tex]

Perimeter = [tex]\sqrt(2426) + \sqrt(2041) + \sqrt(26)[/tex]

Rounding the perimeter to the nearest tenth of a unit:

Perimeter ≈ 49.3 units

Therefore, the value of y is 50, and the perimeter of triangle ABC is approximately 49.3 units.

Learn more about the vertex and area of a triangle.

brainly.com/question/14272448

#SPJ11

A large corporation with monopolistic control in the marketplace has its average daily costs, in dollars, given by C =700 /x + 300x + x^2. The daily demand for x units of its product is given by p = 120,000 - 150 dollars. Find the quantity that gives maximum profit.

Answers

The quantity that gives maximum profit is approximately 111.55 units.

How to calculate the quantity that gives maximum profit

To find the quantity that gives maximum profit, we need to first find the revenue function and then the profit function.

The revenue function is given by:

R(x) = xp = x(120,000 - 150x)

The profit function is given by:

P(x) = R(x) - C(x) = x(120,000 - 150x) - (700/x + 300x + x²)

To find the quantity that gives maximum profit, we need to find the derivative of the profit function and set it equal to zero:

P'(x) = 120,000 - 300x - 700/x² - 2x

Setting P'(x) equal to zero and solving for x, we get:

120,000 - 300x - 700/x² - 2x = 0

Multiplying both sides by x^2, we get:

120,000x² - 300x³ - 700 - 2x³ = 0

Simplifying, we get:

300x³ + 2x³ - 120,000x² - 700 = 0

Dividing both sides by 2, we get:

151x³ - 60,000x² - 350 = 0

Using a graphing calculator or numerical methods, we can find that the real root of this equation is approximately x = 111.55.

Learn more about maximum profit at

https://brainly.com/question/29257255

#SPJ11

Find the length of side x in simplest radical


form with a rational denominator. Xsqrt3

Answers

The length of side x in simplest radical form with a rational denominator is x√3.

To find the length of side x in simplest radical form with a rational denominator given x√3, some steps need to be followed.

Steps are:
1. Identify the radical: In this case, it is √3.

2. Identify the denominator: To rationalize the denominator, we want to eliminate the radical from the denominator. Since the given expression has x√3, the denominator we need to rationalize is 1.

3. Rationalize the denominator: To do this, multiply the expression by a value that will cancel out the radical in the denominator without changing the value of the expression. Since our denominator is 1, we need to multiply the expression by √3/√3.

4. Multiply the expression: (x√3) * (√3/√3) = x√3 * √3 = x(√3)^2 = x(3).

5. Simplify the expression: x(3) = 3x.

So, the length of side x in simplest radical form with a rational denominator is 3x.

To learn more about the “denominator” refer to the https://brainly.com/question/19249494

#SPJ11

The regular price of a sofa, in dollars, is represented by p. the sale price of the sofa is 30% off the regular price. select all true statements. a. the sale price of the sofa can be represented by p-0.3p.

Answers

The sale price of the sofa can be represented by the equation p - 0.3p. This equation correctly represents the sale price after a 30% discount has been applied to the regular price.

The question is about the regular price of a sofa represented by p, and its sale price which is 30% off the regular price. You'd like to know if the statement "the sale price of the sofa can be represented by p-0.3p" is true.

Step 1: Understand the problem
The regular price of the sofa is represented by p. The sale price is 30% off the regular price.

Step 2: Represent the sale price
To find the sale price, we need to subtract the discount (30% of p) from the regular price (p).

Step 3: Calculate the discount
The discount can be calculated as 30% of p, which is 0.3 * p (or 0.3p).

Step 4: Determine the sale price
Now, subtract the discount from the regular price: p - 0.3p.

Step 5: Confirm the statement
The statement "the sale price of the sofa can be represented by p-0.3p" is indeed true.

To know more about the sale price refer to

https://brainly.com/question/7459025

#SPJ11

A sports arena has 40 soda vendors. Each of whom sells 200 sodas per event. Management estimates that for each additional vendor, the yield per vendor decreases by 4. How many additional vendors should management hire to maximize the number of sodas sold.

Answers

Management should hire 25 additional vendors to maximize the number of sodas sold.

Let x be the number of additional vendors that management hires. Then the total number of vendors is 40 + x, and the yield per vendor is 200 - 4x (since the yield decreases by 4 for each additional vendor).

The total number of sodas sold is the product of the number of vendors and the yield per vendor:

Total sodas sold = (40 + x) * (200 - 4x)

To maximize the number of sodas sold, we take the derivative of this expression with respect to x and set it equal to zero:

d/dx [(40 + x) * (200 - 4x)] =

Expanding and simplifying, we get:

-8x² + 120x + 8000 = 0

Dividing both sides by -8, we get:

x² - 15x - 1000 = 0

Using the quadratic formula, we solve for x:

x = (15 ± sqrt(15² + 411000)) / 2

x = (15 ± 35) / 2

x = -10 or x = 25

Since we can't hire a negative number of vendors, the only sensible solution is x = 25. Therefore, management should hire 25 additional vendors to maximize the number of sodas sold.

To learn more about quadratic formula visit: https://brainly.com/question/2615966

#SPJ11

In a circle, the acr length of an intercepted arc is ten inches. The radius of the circle measures 2 inches. What is the measure of the central angle that intercepts that acr?

Answers

The measure of the central angle that intercepts the arc of the circle is approximately 286.48 degrees.

A circle's circumference is comprised of arcs. In other words, if you draw any two locations on a circle's circumference, the curved line that joins them around the border of the circle is referred to as an arc.

The formula for the relationship between arc length and central angle is:

arc length = radius x central angle

We are given the arc length and radius, so we can solve for the central angle:

10 = 2 x central angle

Dividing both sides by 2, we get:

central angle = 10/2

= 5 radians

To convert from radians to degrees, we multiply by 180/π:

central angle = 5 x 180/π

≈ 286.48 degrees.

To know more about a circle, refer:

https://brainly.com/question/24176380

#SPJ4

(answer all for 15 points) 1) Chris went to Waffle House and his bill came out to $8. 23. He had a coupon for 40% off. How much did Leke pay for his meal?


2) To which set or sets does the number -5 belong?


3) Target bought a shirt for $4 from a factory. They mark it up 40%. How much does Target sell the shirt for? $_

Answers

1) Leke pay $4.938 for his meal after a 40% discount.

2) -5 belongs to integers, rational numbers, and real number sets.

3) Target sells the shirt for $5.6 after they mark it up for 40%

1) Bill = 8.23 , discount coupon = 40 %

The total amount paid by leke after the discount coupon = 8.23 - (40 % of 8.23)

Total amount = 8.23 - ( 8.23 × 40/100 )

Total amount = 8.23 - 3.292

The total amount paid by Leke is $4.938

2) -5 is an integer because it is a negative whole number.

-5 is a rational number because it can be written in the form of a fraction.

-5 is a real number because it is a rational number .

-5 belongs to integers, rational numbers, and real number sets.

3) Shirt price = $4 , price increased = 40%

selling price = 4 + (40% of 4)

selling price = 4 + (4 × 40/100)

selling price = 4 + 1.6

Target sell shirt for $5.6

To know more about integers click here :-

https://brainly.com/question/15276410

#SPJ4

A book club of 7 members meet at a local coffee shop. One week, 5 of the
members ordered a small cup of coffee and a muffin. The other 2 members
ordered a small cup of coffee and a piece of banana bread. The cost of a muffin,
including tax, is $3.51. The cost of piece of banana bread is $2. 16 more than
the cup of coffee. The total bill for the book club was $48. 60.

Answers

The cost of a small cup of coffee is $2.97, and the cost of a piece of banana bread is $5.13.

How to solve

Let x represent the cost of a small coffee and y represent the cost of a piece of banana bread. We know:

Cost of muffin: $3.51

y = x + $2.16

5(x + $3.51) + 2(x + y) = $48.60

Substitute y with x + $2.16:

5(x + $3.51) + 2(x + (x + $2.16)) = $48.60

Solve for x:

9x + $21.87 = $48.60

9x = $26.73

x = $2.97

Find y:

y = x + $2.16

y = $2.97 + $2.16

y = $5.13

A slice of banana bread costs $5.13, while a small coffee costs $2.97.

Read more about algebra here:

https://brainly.com/question/432678

#SPJ1

A city is planning a circular fountain, the depth of the fountain will be 3 feet in the volume will be 1800 feet to the third power, find the radius of the fountain, using the equation equals pi to the second power hhhh v is a volume in ours the radius and h is the depth round to the nearest whole number

Answers

The radius of the circular fountain is approximately 17 feet.

The formula for the volume of a circular fountain is given by V = πr^2h, where V is the volume, r is the radius, and h is the depth. In this case, we are given that the depth of the fountain is 3 feet and the volume is 1800 cubic feet. So we can plug in these values into the formula and solve for r as follows:

1800 = πr^2(3)

Simplifying this equation, we get:

r^2 = 600/π

Taking the square root of both sides, we get:

r = sqrt(600/π)

Using a calculator to approximate the value of sqrt(600/π), we get:

r ≈ 17

Therefore, the radius of the circular fountain is approximately 17 feet when rounded to the nearest whole number.

For more questions like Fountain click the link below:

https://brainly.com/question/22563529

#SPJ11

PLEASE HELP!! WILL GIVE BRAINLIEST!!! FIRST ANSWER GETS IT!!

The graph of f(x) and table for g(x)= f(kx) are given.

A coordinate plane with a quadratic function labeled f of x that passes through the points negative 2 comma 4 and negative 1 comma one and vertex 0 comma 0 and 1 comma 1 and 2 comma 4
x g(x)
−2 64
−1 16
0 0
1 16
2 64

What is the value of k?
k = -4
k = 4
k = -1/4

Answers

Answer:

k = 1

Step-by-step explanation:

We can use the table for g(x) = f(kx) to find the value of k.

Notice that when x = -2, we have g(-2) = f(k(-2)) = f(-2k) = 64. Similarly, when x = 2, we have g(2) = f(k(2)) = f(2k) = 64.

Using the fact that f(x) is a quadratic function, we can see that its axis of symmetry passes through the vertex at (0, 0), which means that the x-coordinate of the vertex is 0. This tells us that the coefficient of the x term in f(x) is 0, so the function can be written as f(x) = ax^2 + bx + c, where a is not equal to 0.

Using the points (−2,4), (−1,1), (0,0), (1,1), and (2,4), we can write a system of equations to solve for a, b, and c:

a(-2)^2 + b(-2) + c = 4

a(-1)^2 + b(-1) + c = 1

a(0)^2 + b(0) + c = 0

a(1)^2 + b(1) + c = 1

a(2)^2 + b(2) + c = 4

Simplifying and rearranging, we get:

4a - 2b + c = 4

a - b + c = 1

c = 0

a + b + c = 1

4a + 2b + c = 4

Substituting c = 0 into the system, we get:

4a - 2b = 4

a - b = 1

a + b = 1

4a + 2b = 4

Solving this system of equations, we get a = 1, b = -1, and c = 0.

Substituting these values into g(x) = f(kx), we get:

g(x) = f(kx) = x^2 - x

Substituting the values from the table into this equation, we get:

g(-2) = 4 = (-2)^2 - (-2) = 4k

g(2) = 4 = (2)^2 - (2) = 4k

Solving for k, we get k = 1 or k = -1/4.

However, we need to check which value of k satisfies all the points in between -2 and 2, so we can check g(-1) = 1 = (-1)^2 - (-1) = k, and g(1) = 1 = (1)^2 - (1) = k.

Thus, the value of k that satisfies all the points is k = 1, and therefore the answer is:

k = 1

We can use the information given to find the value of k.

Since the vertex of the quadratic function f(x) is at (0,0), we know that the equation for f(x) is in the form of f(x) = ax^2 for some constant a.

Using the point (-2, 4) on the graph of f(x), we can set up the equation 4 = 4a, which gives us a = 1.

So, the equation for f(x) is f(x) = x^2.

Now, we can use the table for g(x) = f(kx) to find the value of k.

When x = -2, we have g(-2) = f(k(-2)) = f(-2k) = 4k^2.

Similarly, when x = 2, we have g(2) = f(k(2)) = f(2k) = 4k^2.

We also know that g(0) = f(k(0)) = f(0) = 0, and g(-1) = f(k(-1)) = f(-k) = k^2 and g(1) = f(k(1)) = f(k) = k^2.

Using the values from the table, we can set up the following system of equations:

4k^2 = 64

k^2 = 16

0 = 0

k^2 = 16

The only solution that works for all of these equations is k = 4 or k = -4.

Therefore, the value of k is either k = 4 or k = -4.

Scores at a local high school on the Algebra 1 Midterm are extremely skewed left with a mean of 65 and a standard deviation of 8. A guidance counselor takes a random sample of 10 students and calculates the mean score, x¯¯¯.



(a) Calculate the mean and standard deviation of the sampling distribution of x¯¯¯



(b) Would it be appropriate to use a normal distribution to model the sampling distribution? Justify your answer

Answers

The standard deviation of the sampling distribution is 2.53. It would not be appropriate to use a normal distribution to model the sampling distribution.

(a) When dealing with a sampling distribution, the mean of the sampling distribution (μ_x) is equal to the mean of the population (μ). In this case, the mean of the population is 65. Therefore, the mean of the sampling distribution is also 65.

To calculate the standard deviation of the sampling distribution (σ_x), you will use the following formula: σ_x = σ / √n, where σ is the standard deviation of the population and n is the sample size. In this case, the standard deviation of the population is 8 and the sample size is 10. So the standard deviation of the sampling distribution is:

σ_x = 8 / √10 ≈ 2.53

(b) The Central Limit Theorem states that the sampling distribution of the sample mean approaches a normal distribution as the sample size increases, regardless of the shape of the population distribution, provided that the population is not heavily skewed or has extreme outliers. Since the scores are extremely skewed left and the sample size is only 10, it would not be appropriate to use a normal distribution to model the sampling distribution. A larger sample size would be needed to use a normal distribution model.

More on sampling distribution: https://brainly.com/question/16311669

#SPJ11

Can someone help with number 2 pls

Answers

Check the picture below.

[tex]\textit{using the pythagorean theorem} \\\\ a^2+o^2=c^2\implies o=\sqrt{c^2 - a^2} \end{array} \qquad \begin{cases} c=\stackrel{hypotenuse}{13}\\ a=\stackrel{adjacent}{5}\\ o=\stackrel{opposite}{h} \end{cases} \\\\\\ h=\sqrt{ 13^2 - 5^2}\implies h=\sqrt{ 169 - 25 } \implies h=\sqrt{ 144 }\implies h=12 \\\\[-0.35em] ~\dotfill[/tex]

[tex]\textit{volume of a pyramid}\\\\ V=\cfrac{Bh}{3} ~~ \begin{cases} B=\stackrel{base's}{area}\\ h=height\\[-0.5em] \hrulefill\\ B=\stackrel{10\times 10}{100}\\ h=12 \end{cases}\implies V=\cfrac{(100)(12)}{3}\implies V=400~in^3[/tex]

Ms. summers has 1/4 gallon of milk. she drinks 1/8 gallon of the milk and then splits the remaining milk equally between her two children. how much milk does ms. summers give each child? select the expression that could represent the situation

Answers

Ms. Summers gives each child 1/16 gallon of milk.

The amount of milk that Ms. Summers gives to each child can be represented by the following expression:

(1/4 gallon of milk - 1/8 gallon of milk) / 2

This expression represents the amount of milk that remains after Ms. Summers drinks 1/8 gallon of milk, divided equally between her two children.

Simplifying the expression, we get:

(2/8 - 1/8) / 2 = 1/8 / 2 = 1/16

Therefore, Ms. Summers gives each child 1/16 gallon of milk.

To know more about milk refer here

https://brainly.com/question/222134#

#SPJ11

Enter the y coordinate of the solution to this system of equations. 3x+y=-2 x-2y=4

Answers

The y coordinate of the solution to this system of equations is -2

Calculating the y coordinate of the solution to this system of equations.

From the question, we have the following parameters that can be used in our computation:

3x+y=-2 x-2y=4

Express properly

So, we have

3x + y = -2

x - 2y = 4

Multiply the second equation by -3

so, we have the following representation

3x + y = -2

-3x + 6y = -12

Add the equations to eliminate x

7y = -14

Divide both sides by 7

y = -2

Hence, the value of y is -2

Read more about system of equations at

https://brainly.com/question/13729904

#SPJ1

30. Mean IQ of Attorneys See the preceding exercise, in which we can assume that o = 15


for the IQ scores. Attorneys are a group with IQ scores that vary less than the IQ scores of the


general population. Find the sample size needed to estimate the mean IQ of attorneys, given that


we want 98% confidence that the sample mean is within 3 IQ points of the population mean.


Does the sample size appear to be practical?

Answers

A sample size of 40 attorneys is needed to estimate the mean IQ with 98% confidence and a margin of error of 3 IQ points.

To find the sample size, we use the formula:

n = (z*σ/E)²

where n is the sample size, z is the z-score for the desired level of confidence given as 98% , σ is the population standard deviation given as 15, and E is the margin of error given as 3 IQ points.

using the above values, we get:

n = (2.33*15/3)²

n = 39.05

Therefore, we need a sample size of at least 40 attorneys to estimate the mean IQ with 98% confidence and a margin of error of 3 IQ points.

Whether this sample size is practical or not depends on various factors, such as the availability of attorneys with the desired characteristics, the cost and time required to collect the data, and the resources available for analysis. In general, a sample size of 40 is considered moderate to large for many applications, and it may be feasible depending on the specific context.

A sample size of 40 attorneys is needed to estimate the mean IQ with 98% confidence and a margin of error of 3 IQ points.

To know about Sample size here

https://brainly.com/question/25894237

#SPJ4

Section 15 8: Problem 3 Previous Problem Problem List Next Problem 3 (1 point) Find the maximum value of f(x, y) = xºy® for x, y > 0 on the unit circle. = fmax

Answers

The maximum value of f(x, y) = x^y on the unit circle can be found using the constraint x^2 + y^2 = 1, which defines the unit circle. To solve this, we can use the method of Lagrange multipliers.

Let g(x, y) = x^2 + y^2 - 1. Then, the gradient of f(x, y) and the gradient of g(x, y) should be proportional:
∇f(x, y) = λ∇g(x, y)

Calculating the gradients:
∇f(x, y) = (yx^(y-1), x^y * ln(x))
∇g(x, y) = (2x, 2y)

Equating the components and dividing the equations, we get:
y * x^(y-1) / 2x = x^y * ln(x) / 2y

Simplifying, we obtain:
ln(x) = y

Now, using the constraint x^2 + y^2 = 1, we can substitute y with ln(x) and solve for x:
x^2 + (ln(x))^2 = 1

Numerically solving this equation, we get x ≈ 0.90097 and y ≈ ln(0.90097) ≈ -0.10536. Since we are only interested in positive values of x and y, this is the only solution in our domain. Now, we can find the maximum value of f(x, y):
f_max = f(0.90097, -0.10536) ≈ 0.79307

So the maximum value of f(x, y) on the unit circle is approximately 0.79307.

Learn more about maximum value, here:

brainly.com/question/24866444

#SPJ11

A parabola can be drawn given a focus of (−4,5) and a directrix of y=−9. what can be said about the parabola?

Answers

The parabola with a focus of (-4, 5) and a directrix of y = -9 is vertically oriented, opens upward, has a vertex at (-4, -2), and its equation is (x + 4)^2 = 28(y + 2).

1. The parabola is vertically oriented since the directrix is a horizontal line.
2. The vertex of the parabola is equidistant from the focus and the directrix. To find the vertex, we can calculate the midpoint between the focus and a point on the directrix with the same x-coordinate: (-4, -9 + (5 - (-9))/2) = (-4, -9 + 7) = (-4, -2).
3. The parabola opens upward because the focus is above the directrix.
4. The equation of the parabola can be found using the vertex form: (x - h)^2 = 4p(y - k), where (h, k) is the vertex and p is the distance between the vertex and the focus or directrix. In this case, (h, k) = (-4, -2), and p = (5 - (-2)) = 7. The equation is therefore (x + 4)^2 = 28(y + 2).

In summary, the parabola with a focus of (-4, 5) and a directrix of y = -9 is vertically oriented, opens upward, has a vertex at (-4, -2), and its equation is (x + 4)^2 = 28(y + 2).

learn more about "Parabola":-https://brainly.com/question/4061870

#SPJ11

Other Questions
People can influence the environments in which plants grow in many different ways. some of these influences can prevent disease and some can encourage disease. discuss an example of each situation. principal: $5,000, annual interest: 6%, interest periods: 12, number of years: 18After 18 years, the investment compounded periodically will be worth $(Round to two decimal places as needed.)more than the investment compounded annually. Hammerin' hanklook at the timeline based on the passage. which detail belongs in box #1 Ted Pappas pays$4,388. 65 in real estate taxes yearly. His property has a market value of$119,340. 00 with a rate of assessmentof 42%. What is his tax rate to thenearest tenth of a mill? please help as soon as possible A waitress sold 11 ribeye steak dinners and 14 grilled salmon dinners, totaling $551.11 on a particular day. Another day she sold 15 ribeye steak dinners and 7 grilled salmon dinners, totaling $581.47. How much did each type of dinner cost? Where should one start to learn maths if they're really bad at it What emotion are teen mothers most likely to experience?A. excitementB. surpriseC. depressionD. happinessE. calmness There are many things happening on this graph, but you only need to focus on the green stuff. What conclusion can you draw from the green data?Question 8 options:No sunspots were observed during the middle ages (Years 1100-1600)Sunspot data from the middle ages is much more detailed than sunspot data from 1700-1900.Sunspot data from 1700-1900 is much more detailed than sunspot data from the middle ages.Every person who collected this data is still alivethis is earth science The following question references the novel the three musketeers by alexandre dumas.there are several affairs that take place throughout the course of the novel. consider the marriage of the king and queen, monsieur and madame bonacieux, and athos and milady. what is the view of marriage in the three musketeers? write at least three sentences. the questio is write a rule to describe each transformation please please help me Helpppppppp(serious answers only) THE PUZZLEThis problem gives you the chance toSolve and reason abxut equationsA magazine contains a puzzle. Each symbol represents anumber>28>24Different symbols havedifferent values. N42The sum of each row is givenat the side of the table. >36Try to find out the value for each symbol:heart-. Spade - 1. Club - 4 diamond Which of the following statements is true about the Patients' Bill of Rights?The Patients' Bill of Rights is the fifteenth amendment to the U.S. Constitution.No one universal government statute exists.The Patients' Bill of Rights is part of HITECH.The American Hospital Association has a Patients' Bill of Rights that became federal law. what is the most famous of his political paintings depicting the revenge of the french firing squad? which biome is known for being hot and moist with constant rain A sample of an ideal gas has a volume of 2.31 L at 279 K and 1.01 atm. Calculate the pressure when the volume is 1.09 L and the temperature is 308 K. On December 1, NNC began an electro repair shop,Samson Quality Automotive. The following transactions occurred during December Dec. 1 Samson contributed birr 55,000 cash to the business in exchange for shares of common stock. 1 Purchased birr 9,000 of equipment paying cash. 1 Paid birr 1,500 for a five-month insurance policy starting on December 1. 9 Paid birr 16,000 cash to purchase land to be used in operations. 10 Purchased office supplies on account, birr 2,000. 19 Borrowed birr 12,000 from the bank for business use. Samson signed a notes payable to the bank in the name of the corporation. The note is due in five years. 22 Paid birr 1,700 for advertising expenses. 26 Paid birr 300 on account. 28 The business received a bill for utilities to be paid in January, birr 180. 31 Revenues earned during the month included birr 14,000 cash and birr 2,600 on account. 31 Paid employees salaries birr 3,800 and building rent birr 1,300. Record as a compound entry. 31 The business received birr 1,400 for auto screening services to be performed next month. 31 Cash dividends of birr 4,000 was paid to stockholders. Let and be two ordered bases of , and consider a linear transformation. Suppose that the change of base matrix is given by and the coordinate matrix of with respect to is given by use this to determine coordinate matrix of with respect to. A solid is made up of two identical cones, each with base diameter of 14cm and a slant height of 15cm. Find its Volume.